1answer.
Ask question
Login Signup
Ask question
All categories
  • English
  • Mathematics
  • Social Studies
  • Business
  • History
  • Health
  • Geography
  • Biology
  • Physics
  • Chemistry
  • Computers and Technology
  • Arts
  • World Languages
  • Spanish
  • French
  • German
  • Advanced Placement (AP)
  • SAT
  • Medicine
  • Law
  • Engineering
oksian1 [2.3K]
3 years ago
13

Find the measure of \angle G∠G. Round your answer to the nearest tenth (one decimal place).

Mathematics
1 answer:
Law Incorporation [45]3 years ago
6 0

Hi there!

\large\boxed{G \approx 55.44^{o}}

To solve, we can use right triangle trigonometry.

Recall that:

sin = O/H, cos = A/H, tan = O/A.

For angle G, HF is its OPPOSITE side, and FG is the hypotenuse.

Therefore, we must use sine to evaluate:

sinG = 14 / 17

sin⁻¹ (14/17) = ∠G. Evaluate using a calculator.

∠G ≈ 55.44°

You might be interested in
(Please answer quickly)
Elenna [48]

Answer:

huge erection

Step-by-step explanation:

4 0
3 years ago
Read 2 more answers
Given that events A and B are mutually exclusive, i.e., A?B = ? and P(A?B) =0, and that P(A)=0.7 and P(B)=0.2, find P(A or B).
TEA [102]

Answer:

B

Step-by-step explanation:

i use d  relationship between them

7 0
3 years ago
Which of these is a linear equation in<br> standard form for the graph shown?
vampirchik [111]

Answer:

B

Step-by-step explanation:

Because the slope is a positive slope (goes through the positive quandrants on the graph), we can eliminate answer choices c. Now, if we solve a, b, and d and we can see if the slope + y-intercept matches the graph.

For A.) (solve for y to get a y=mx + b equation)

3x-2y=4

-2y= -3x + 4

<u>y= 3/2 -2</u>

For B.)

3x-2y= -4

-2y= -4-3x

<u>y= 3/2 + 2</u>

For D.)

2x+3y= 4

3y= 4-2x

<u>y= -2/3x + 4/3</u>

Now, if we look at D, the slope is negative so it cannot be D. That leaves us with A and B. The b in the y=mx + b equation stands for the y-intercept, so we can see that the y-intercept is 2. So, since the graph intersects the y-axis at (0,2), the answer is B! y=3/2 + 2 has a positive slope and 2 for the y-intercept!

4 0
3 years ago
After a 12% increase the cost of a new cooker is £190.40
Svetllana [295]

Answer:

1

X+.12*x=190.40

X(1+.12)=190.40

X=190.40/1.12=170

2a

X+x+25=100

2x=75

X=37.5%

Men=.375*270=101

2b

4 0
2 years ago
The reduced cost for a decision variable that appears in a Sensitivity Report indicates the change in the optimal objective func
pshichka [43]

Answer:C

Step-by-step explanation:

Looking at number line, the function value always range from positive value from 0 to negative numbers to -1

8 0
3 years ago
Read 2 more answers
Other questions:
  • in the figure below , Line K is perpendicular to Line l. based on the information given in the figure, find the measure of &lt;C
    12·2 answers
  • A chef uses a recipe with the following ingredients. Then the chef decides to make more than 1 batch of the recipe. Fruit Cups U
    10·2 answers
  • help! A loaf of bannana bread serves 6 guests. There will be 47 guests attending the faculty breakfast. Which expression shows h
    14·1 answer
  • At the age of 20 weeks, Zoe weighed 16 lb 4 oz. Her birth weight was about 33 1/3% of her 20- week weight. To the nearest ounce,
    5·1 answer
  • Original Price<br> Percent of Discount<br> Sale Price<br> $82<br> $65.60
    5·1 answer
  • 4. Nick drained 2.8 gallons of water from a fish
    10·2 answers
  • How to find area of a triangle inside a rectangle
    9·2 answers
  • I need help thank you
    6·1 answer
  • PLEASE HELP!! (Look at the photo)
    12·2 answers
  • Estimates by first rounding each number to the place value 1.8×3.62
    9·1 answer
Add answer
Login
Not registered? Fast signup
Signup
Login Signup
Ask question!